LSAT 9 – Section 4 – Question 21

You need a full course to see this video. Enroll now and get started in less than a minute.

Target time: 1:04

This is question data from the 7Sage LSAT Scorer. You can score your LSATs, track your results, and analyze your performance with pretty charts and vital statistics - all with a Free Account ← sign up in less than 10 seconds

Question
QuickView
Type Tags Answer
Choices
Curve Question
Difficulty
Psg/Game/S
Difficulty
Explanation
PT9 S4 Q21
+LR
Strengthen +Streng
A
2%
158
B
3%
159
C
2%
156
D
5%
162
E
88%
165
126
139
152
+Easier 146.711 +SubsectionMedium
This page shows a recording of a live class. We're working hard to create our standard, concise explanation videos for the questions in this PrepTest. Thank you for your patience!

This is a strengthening question, though it may be difficult to identify. We should know it is a strengthening question because the stem asks: On the basis of the premises advanced, which one of the following principles, if established, would provide the most justification for the concluding recommendation?

The stimulus begins with the statistic that 99% of the burglar alarms police respond to are false alarms. Seems like a big waste of time! The stimulus explicitly tells us that this is a drain on important resources, with each false alarm wasting an average of ~45 minutes of police time. This draws police attention away from other crimes and mostly benefits businesses and the rich, but on the other hand alarm systems are effective deterrents of burglaries. The author concludes by recommending a compromise: police keep responding to burglar alarms but fine alarm owners the cost for false alarms. Our job is to find the principle in the answer choices will justify the author’s recommendation. Let’s see our options:

Answer Choice (A) This principle would justify eliminating burglar alarm systems, not the author’s compromise.

Answer Choice (B) The author is recommending that burglar systems owners be fined in specifically the case of false alarms, regardless of whether police resources are in short supply or they can afford to.

Answer Choice (C) We haven’t been told anything about whether the author’s recommendation would reduce crime levels throughout the entire area the police serve, just that they are deterrent for their owners!

Answer Choice (D) Important to remember the condition for people being required to pay is if they waste police time with a false alarm, not whether they directly benefit from the police service

Correct Answer Choice (E) We’ve been explicitly told that false alarms waste police time which could be spent on other legitimate calls, so this would justify the author’s recommendation that false alarm owners be fined.

Take PrepTest

Review Results

Leave a Reply